LSAT 1 – Section 4 – Question 12
You need a full course to see this video. Enroll now and get started in less than a minute.
Target time: 0:52
This is question data from the 7Sage LSAT Scorer. You can score your LSATs, track your results, and analyze your performance with pretty charts and vital statistics - all with a Free Account ← sign up in less than 10 seconds
Question QuickView |
Type | Tags | Answer Choices |
Curve | Question Difficulty |
Psg/Game/S Difficulty |
Explanation |
---|---|---|---|---|---|---|---|
PT1 S4 Q12 |
+LR
| Weaken +Weak | A
59%
163
B
3%
155
C
14%
151
D
24%
154
E
1%
134
|
142 154 166 |
+Harder | 144.766 +SubsectionEasier |
This is a weakening question, as the question stem asks: Which one of the following, if true, casts the most doubt on the author’s hypothesis?
This is a fairly straight forward correlation-causation argument. A study finds that smokers are more likely to snore, and concludes that smoking can cause snoring. Our job is to weaken this argument. Let’s see what we get:
Correct Answer Choice (A) This provides an explanation for why snoring and smoking would correlate, even when smoking might have no influence on snoring, namely that the two share a third cause, stress.
Answer Choice (B) Ok, but unless we know obesity leads to snoring this does nothing for us.
Answer Choice (C) This is completely compatible with the argument.
Answer Choice (D) Same as C.
Answer Choice (E) Interesting! But this could be true while smoking caused snoring, and does nothing to suggest it doesn’t.
Take PrepTest
Review Results
LSAT PrepTest 1 Explanations
Section 1 - Reading Comprehension
- Passage 1 – Passage
- Passage 1 – Questions
- Passage 2 – Passage
- Passage 2 – Questions
- Passage 3 – Passage
- Passage 3 – Questions
- Passage 4 – Passage
- Passage 4 – Questions
Section 2 - Logic Games
Section 3 - Logical Reasoning
- Question 01
- Question 02
- Question 03
- Question 04
- Question 05
- Question 06
- Question 07
- Question 08
- Question 09
- Question 10
- Question 11
- Question 12
- Question 13
- Question 14
- Question 15
- Question 16
- Question 17
- Question 18
- Question 19
- Question 20
- Question 21
- Question 22
- Question 23
- Question 24
- Question 25
Leave a Reply
You must be logged in to post a comment. You can get a free account here.